User avatar
 
geverett
Thanks Received: 79
Atticus Finch
Atticus Finch
 
Posts: 207
Joined: January 29th, 2011
 
 
 

Q23 - Editorial: This political party

by geverett Wed May 25, 2011 9:12 pm

I can see that A is wrong, because it is stated explicitly in the stimulus and so is not an assumption but answer choice D seems too broad to be necessary and seems to qualify more as a sufficient assumption. Anybody have any ideas?

I think I might have just figured this one out myself. Try getting rid of the double negative in answer choice D, and it might become clearer. However, for the sake of discussion and since there is not post on this question I'm going to let the conversation roll out anyways to hear other people's thoughts on this.
User avatar
 
bbirdwell
Thanks Received: 864
Atticus Finch
Atticus Finch
 
Posts: 803
Joined: April 16th, 2009
 
This post thanked 1 time.
 
 

Re: Q23 - Editorial: This political party

by bbirdwell Sat May 28, 2011 4:45 am

Here's my two cents.

Education is a worthy goal
Govt should not increase spending on education
Conclusion: above is inconsistent.

Hmm. Well, it doesn't seem inconsistent because "education" and "spending" are different things. The only way this argument works is if those two concepts are connected somehow.

(A) close! But voting isn't part of it. The fact that it's about a legislator rather than party policy is the red flag that brought my attention to this. "Claiming the govt should spend" and "voting against an increase" are two totally different things.

(B) out of scope! Not even close.

(C) way out scope. way, way out.

(D) i like it already because it's about policy, but dude! That double negative is gnarly! And "perform" and "spending" don't seem like a great match. I'd leave it for now and check (E).

(E) close again, but not the same thing. Try negating it. Even if the members of the party always have inconsistent views, this says nothing about the policies of the party itself being inconsistent.

So, at this point, I check the clock. If I'm feeling the squeeze, I make the LSAT decision to choose (D) because it's the best ballpark match. If I have a little time, I take a second look at it.

It seems to say: Consistent --> ~(worthy -->~performed)

My conclusion includes "NOT consistent," so I'm gonna contrapose that statement and see what I get.

(worthy --> not performed) --> inconsistent

Whoa! If I can stand to let "performance" and "spending" be roughly synonymous, this is exactly what the argument is saying. It's the invisible connection that between the two facts and the conclusion that they are inconsistent. Bingo.

Quick negation thinking tells me that if this choice is not true, then the argument's conclusion cannot be drawn, period. That indicates a necessary assumption.

Maybe that helps?
I host free online workshop/Q&A sessions called Zen and the Art of LSAT. You can find upcoming dates here: http://www.manhattanlsat.com/zen-and-the-art.cfm
 
jamiejames
Thanks Received: 3
Atticus Finch
Atticus Finch
 
Posts: 116
Joined: September 17th, 2011
 
 
 

Re: Q23 - Inconsistent position on education spending

by jamiejames Wed Feb 08, 2012 7:53 pm

this is sadly one of those situations where all the answers seem a little wrong, and you have to choose the best of a bad bunch.

(a) talks about legislators and voting, which aren't brought up in the stimulus.

(b) is out of scope

(c) is out of scope.

(d) this one has a double negative, so it's a little bit of a pain, but keep it.

(e) The stimulus is talking about the party as a whole, not individual members. Also, "never" is too strong of a word to use. There's just too much wrong with this answer for it to be right.

So already, we've knocked down 4/5 answers, and in a crunch, I always go with the answer that looks kind of right, especially when I know for a fact that the other 4 are wrong.
 
MayMay
Thanks Received: 0
Forum Guests
 
Posts: 25
Joined: January 02nd, 2013
 
 
 

Re: Q23 - Editorial: This political party

by MayMay Sun Jan 06, 2013 6:43 pm

maybe i'm being too picky but to me, this stimulus tell ms what's inconsistent. it doesn't tell me anything about what's consistent.
So even though answer choice D is the best of the five, i'm still not happy with it.
what do you guys think?
User avatar
 
bbirdwell
Thanks Received: 864
Atticus Finch
Atticus Finch
 
Posts: 803
Joined: April 16th, 2009
 
 
 

Re: Q23 - Editorial: This political party

by bbirdwell Tue Jan 08, 2013 2:13 pm

See my explanation above. Inconsistent becomes consistent when it's contraposed.
I host free online workshop/Q&A sessions called Zen and the Art of LSAT. You can find upcoming dates here: http://www.manhattanlsat.com/zen-and-the-art.cfm
 
rbkfrye
Thanks Received: 0
Forum Guests
 
Posts: 10
Joined: February 22nd, 2013
 
 
 

Re: Q23 - Editorial: This political party

by rbkfrye Tue May 21, 2013 7:40 am

I considered the contra approach, but rejected it as the contrapositive's necessary clause is technically not "inconsistent", but rather "not consistent", reasoning that things that aren't consistent can also be not inconsistent. In retrospect, I probably took the LGB's contrapositive rules a little too literally...
User avatar
 
WaltGrace1983
Thanks Received: 207
Atticus Finch
Atticus Finch
 
Posts: 837
Joined: March 30th, 2013
 
 
trophy
Most Thanked
trophy
Most Thankful
trophy
First Responder
 

Re: Q23 - Editorial: This political party

by WaltGrace1983 Mon May 05, 2014 12:15 pm

Almost fell for the trap answer, (A)! Whew. Looking at it again though, wouldn't (A) be right if a slight modification was made?

Here is (A) as is...

    (Claim Worthy Goal & ~Vote For It & Legislator) → Inconsistent


I think this language is right. "It is inconsistent for..." sets up the necessary condition. What is it talking about? It is talking about "legislator," "claiming that increasing spending on education is a worthy goal," and "vote against increasing spending on education." That is basically EXACTLY what the argument says. The only problem? The "legislator" part. We know it is inconsistent for a party's policy, but we don't exactly know if it is for a legislator. Tricky!

A true necessary condition would just ditch the "legislator" part and that would make it correct, no?
 
cwolfington
Thanks Received: 4
Jackie Chiles
Jackie Chiles
 
Posts: 29
Joined: May 15th, 2014
 
 
 

Re: Q23 - Editorial: This political party

by cwolfington Wed Jun 11, 2014 2:21 am

D is correct because the opposite of consistent is inconsistent. Yes it's a polar opposite, but what is the logical opposite of consistent- not consistent, which is the same as inconsistent.

If you negate D it becomes, "A consistent political policy holds that an action that compromises a worthy goal should not be performed." This destroys the argument because it would mean that the political party is consistent, whereas the argument states the opposite.
 
cynthiaemesibe
Thanks Received: 1
Vinny Gambini
Vinny Gambini
 
Posts: 6
Joined: October 13th, 2015
 
 
 

Re: Q23 - Editorial: This political party

by cynthiaemesibe Tue Jul 12, 2016 12:09 pm

It was between A and D for me and I ultimately ended up going with A, however even with choosing A I didn't feel it was the perfect answer, just better than D. I see why A is wrong, but my problem with answer choice D is it states what a consistent party is. We don't necessarily know what the editorial believes to be consistent just what he believes to be inconsistent.
 
nworden4band
Thanks Received: 0
Vinny Gambini
Vinny Gambini
 
Posts: 1
Joined: August 24th, 2016
 
 
 

Re: Q23 - Editorial: This political party

by nworden4band Wed Aug 24, 2016 4:08 pm

To anyone who is still scratching their heads about this and thinking that it seems to be the opposite of what's said on this forum, I think you may have made the same mistake that I made.

You probably think the stimulus say compromise and not comprise.

Well my friends, the stimulus actually says comprise. C-O-M-P-R-I-S-E. Like an automobile is comprised of parts. Or a cake comprised of various ingredients. I'm a contextual reader, and this drove me mad.

So endeth the lesson
 
JosephV
Thanks Received: 9
Jackie Chiles
Jackie Chiles
 
Posts: 38
Joined: July 26th, 2017
 
 
 

Re: Q23 - Editorial: This political party

by JosephV Wed Jan 24, 2018 2:09 pm

nworden4band Wrote:To anyone who is still scratching their heads about this and thinking that it seems to be the opposite of what's said on this forum, I think you may have made the same mistake that I made.

You probably think the stimulus say compromise and not comprise.

Well my friends, the stimulus actually says comprise. C-O-M-P-R-I-S-E. Like an automobile is comprised of parts. Or a cake comprised of various ingredients. I'm a contextual reader, and this drove me mad.

So endeth the lesson


A point of clarification.

You are not talking about the stimulus but about the correct answer choice (C) which reads: "A consistent political policy does not hold that an action that comprises a worthy goal should not be performed."